shoki

New Member
ارسال ها
637
لایک ها
128
امتیاز
0
#41
حالا مسئله ی بعدی :
یه مثلث ABC داریم و سه نقطه ی X,Y,Z روی اضلاع BC,CA,AB هستند به طوری که AX,BY,CZ همرسند.محل تماس دایره ی محاطی داخلی با اضلاع BC,CA,AB رو D,E,F بنامید. حالا از هر کدوم از نقاط X,Y,Z مماس های XP,YQ,ZR رو بر دایره ی محاطی داخلی که مرکزش I هست رسم کنید. دقت کنید که P,Q,R به غیر از D,E,F می باشند .حالا ثابت کنید که AP,BQ,CR همرسند.
 
ارسال ها
62
لایک ها
1
امتیاز
0
#42
خوب این اسونه:
قطبهای AX,... به ترتیب FE اشتراک PD ,مشابه برای بقیه هستند, AP,مشابها همرس اند چون نقاط اشتراک قبلی همخط اند (قطبهای AX ,...)
اگه درسته بعدی رو بدم؟
 

shoki

New Member
ارسال ها
637
لایک ها
128
امتیاز
0
#43
میتونی ویرایشش کنی چون که همون اول کار یه اشکال تایپی داری!! قطب AD می شه محل برخورد EF با BC !!
 

shoki

New Member
ارسال ها
637
لایک ها
128
امتیاز
0
#45
ehh ... به نظر میاد یه تیکه ی اثبات رو جا انداخته باشی !! اون اول کار یعنی :
'' قطبهای AX,... به ترتیب FE اشتراک PD ,مشابه برای بقیه هستند'' و اون آخر کار یعنی :
''
نقاط اشتراک قبلی همخط اند (قطبهای AX ,...) '' هم درسته ... ولی این وسط کار یعنی : ''
AP ,مشابها همرس اند '' رو کامل اثبات نکردی چون که باید ثابت کنی که قطب های AP,BQ,CR
هم خطند.
 
ارسال ها
62
لایک ها
1
امتیاز
0
#46
نه.من از اون لم تو دایره محاطی استفاده کردم که همرسی را به یک همرسی دیگر تبدیل میکند...
 

shoki

New Member
ارسال ها
637
لایک ها
128
امتیاز
0
#47
می تونی اونو لم رو بگی !
 
ارسال ها
62
لایک ها
1
امتیاز
0
#48
در مثلث ABC دایره محاطی AB,AC,BC را در Z,Y,X طع میکند.اگر M,N,T روی دایره محاطی باشند AM,BN,CT همرس اند اگر و تنها اگر XM,NY,ZT همرس باشند یا XM شتراک ZY ,مشابها همخط باشند.(اینجا همخطی استفاده شده)
 

shoki

New Member
ارسال ها
637
لایک ها
128
امتیاز
0
#50
آره درسته من این لم رو می دونستم ولی در مورد هم خطیش فکر نکرده بودم که ببینم درسته یا نه !! با این اوصاف، اثبات من دقیقا میشه اثبات لمی که دادی !!
@Goharshady :
in ghadar tashakor nakon !!!
asabamo dari khurd mikoniya !!
 

shoki

New Member
ارسال ها
637
لایک ها
128
امتیاز
0
#52
ابتدا فرض کنید که
نقاط تماس دایره با
باشد. قرار دهید :
و به همین ترتیب 'B',C',D رو تعریف کنید.
شعاع دایره رو r قرار دهید، حالا داریم :

دقت کنید که از این نکته استفاده شد که AA'D'D محاطی هست که اثباتش هم راحته ...
حالا همین نسبت ها رو برای باقی کسر ها می نویسیم. از طرفی هم داریم :​

از طرفی دقت کنید که 'A'IB و 'D'IC مکمل همند و بنابریان سینوس هاشون با هم برابره. و به همین ترتیب برای
. پس در کل باید یه چنین عبارتی رو ثابت کنیم :


حالا به جای
می تونیم بنویسیم :
و با جاگذاریهای مشابه به این تساوی می رسیم :

( به جای ... مقادیر مشابه
می باشند) . حالا برای اثبات این تساوی هم کافیه که این جای گذاری رو در نظر بگیرید :
و
و
و
و در نظر گرفتن این دو نکته که :
و
. دیگه باقیش با همین جایگذاری ها حله ... در واقع کافیه که سمت راست رو کاملا بسطش بدید . بعدش دو طرف عین هم می شند.​
 

shoki

New Member
ارسال ها
637
لایک ها
128
امتیاز
0
#53
سوال بعدی که در مورد Fuhrmann's theorem هست .
1- Fuhrmann's theorem رو اثبات کنید:
ثابت کنید قرینه های اوساط کمان های BC,CA,AB ( که شامل A,B,C نمی باشند ) نسبت به اوساط اضلاع BC,CA,AB هم دایره با مرکز ارتفاعی ABC می باشند.
2- Generalization of Fuhrmann's theorem
P یک نقطه ی دلخواه است.فرض کنید که AP,BP,CP دایره ی محیطی ABC رو در 'A',B',C قطع می کنه. ثابت کنید قرینه های 'A',B',C نسبت به اوساط اضلاع BC,CA,AB با مرکز ارتفاعی ABC هم دایره هستند. دقت کنید که در قسمت قبلی P=I بود که I مرکز دایره ی محاطی داخلی هست.
 
ارسال ها
62
لایک ها
1
امتیاز
0
#54
من راه سوال قبلیتون را کامل نخوندم ولی به نظر میرسه ساده فکر نکردین.کافی بود M را روی AB جدا کنید که BMI=BIC (angles) ...
 
ارسال ها
62
لایک ها
1
امتیاز
0
#55
فکر کنم 2 را اشتباه نوشتی.باید نسبت به ضلع قرینه کنیم نه وسط.
 

shoki

New Member
ارسال ها
637
لایک ها
128
امتیاز
0
#56
نه، درسته
در واقع هم اگه قرینه کنیم نسبت به ضلع درسته و هم اگه قرینه کنیم نسبت به وسط درسته ...
اگه تونستید این رو هم اثبات کنید !
 

shoki

New Member
ارسال ها
637
لایک ها
128
امتیاز
0
#58
اگه 1 و 3 رو اثبات کردی حتما می تونی 2 رو هم اثبات کنی !! good luck
 
ارسال ها
62
لایک ها
1
امتیاز
0
#59
یه راه فضایی:
2)انعکاس به مرکز H.باید ثابت کنیم منعکس قرینه ها همخط اند چون قرینه ها روی دایره های AHB,AHC,BHC هستند(بدیهیه!).سوا سینوسی قبل انعکاس منلاووس این همطی رو میده...حل شد.
1)این دایره به قطر HN_a (ناگل) است.
سوال بعد :ثابت کنید I مرکز ارتفاعی مثلث به وجود امده در 1 است.
 

shoki

New Member
ارسال ها
637
لایک ها
128
امتیاز
0
#60
اون وقت نسبت انعکاس چند باید باشه ؟؟؟ دلخواه؟ خواهشا اگه می شه راه حلاتو کامل تر بنویس

در مورد سوال 1 و 2 راه حل من همون طور که بعدا در یک لینک بدان اشاره خواهد شد با استفاده از مختصات (x,y) هست و اگر هم کسی بخواد راه حلمو کامل می نویسم ( بر خلاف اون چیزی که شاید اول کار آدم تصور کنه، راه حلش با مختصات (x,y) اصلا طولانی نیست بلکه کوتاه تر از هر راه حلی هم هست چون در واقع ( به صورت هوشمندانه
) فقط قسمتی از شکل مورد بررسی قرار می گیره!) .
در مورد سوال 3 ( قرینه نسبت به اضلاع ) : می شه با کمی بررسی فهمید که مسئله ی 2 و3 به هم تبدیل می شند!
و اما در مورد سوالی که تو گذاشتی باید بگم که حالت کلیش هم درسته !!!

خیلی دوست دارم ببینم راه حلت برای این یکی چیه !!!
ثابت کنید که وقتی اون نقاط رو نسبت به ضلع قرینه می کنیم مثلثی که به وجود میاد با مثلث 'A'B'C متشابه هست و نقطه ی P به گونه ای هست که در هر دو مثلث نظیر هم هست ( یعنی نقطه ی نظیر P در مثلث 'A'B'C همون P می شه ! )... در حالت خاص ( یعنی همون سوال 1 ) اگر دقت کنید اون دو مثلث متشابه اند و نقطه ی I مرکز ارتفاعی هر دوی آن هاست! یعنی نظیر نقطه ی I در 'A'B'C خودش می شه!)
در نهایت باید بگم این نکته که قطر دایره هم در مسئله ی 1 ، HN_a هست چیز جالبی بود که من تا حالا ازش غافل بودم!
 
بالا